Đến nội dung

chardhdmovies

chardhdmovies

Đăng ký: 25-04-2014
Offline Đăng nhập: 27-07-2016 - 14:24
****-

#616177 Tìm số cách tô màu thỏa đề.

Gửi bởi chardhdmovies trong 21-02-2016 - 09:00

Cho đa giác đều $n$ đỉnh nội tiếp đường tròn tâm $O$, tìm số cách tô $n$ đỉnh tam giác bằng $m (m\geq2)$ màu sao cho 2 đỉnh kề nhau không được tô cùng màu. (Hai cách tô được coi là giống nhau nếu chúng nhận được qua nhau bằng một phép quay tâm $O$)




#614806 $\frac{x^4}{(x^2+y^2)(x+y)}$

Gửi bởi chardhdmovies trong 13-02-2016 - 23:16

Cho $x,y,z>0$ thoả mãn $x+y+z=1$. Tìm $min$ của $F=\frac{x^4}{(x^2+y^2)(x+y)}+\frac{y^4}{(y^2+z^2)(y+z)}+\frac{z^4}{(z^2+x^2)(z+x)}$
tks you =))

Gợi ý: Chứng minh $\sum{\frac{x^4}{(x^2+y^2)(x+y)}}=\sum{\frac{y^4}{(x^2+y^2)(x+y)}}$




#606966 TOPIC ôn thi Olimpic 30/04 và thi HSG toán 10

Gửi bởi chardhdmovies trong 03-01-2016 - 15:46

Nhiệt liệt ủng hộ topic này luôn, chúc topic phát triển và đóng góp nhiều bài tập hay  :lol:

Bài 2: Cho a, b, c, d là các số thực dương thỏa mãn đk: $\frac{1}{a} + \frac{1}{b} + \frac{1}{c} + \frac{1}{d} = 4$. CM:

$\sqrt[3]{\frac{a^{3} + b^{3}}{2}} + \sqrt[3]{\frac{b^{3} + c^{3}}{2}} + \sqrt[3]{\frac{c^{3} + d^{3}}{2}} + \sqrt[3]{\frac{d^{3} + a^{3}}{2}} \leq 2(a + b + c + d) - 4$

Bài 3: CMR nếu a,b,c>0 thì: 

$\frac{(a + b + c)^{2}}{ab + bc + ca} \geq \frac{a + b}{a + c} + \frac{b + c}{b + a} + \frac{c + a}{c + b}$

3. Giả sử $c=min\left \{ a,b,c \right \}$. Ta biến đổi BĐT như sau:

$\frac{(a+b+c)^{2}}{ab+bc+ca}-3\geq (\frac{a+b}{a+c}-1)+(\frac{b+c}{b+a}-1)+(\frac{c+a}{c+b}-1)$

$\Leftrightarrow \frac{a^2+b^2+c^2-ab-bc-ca}{ab+bc+ca}\geq \frac{b-c}{a+c}+\frac{c-a}{b+a}+\frac{a-b}{c+b}$

$\Leftrightarrow \frac{(a-b)^2+(a-c)(b-c)}{ab+bc+ca}\geq\frac{(a-c)(b-c)+b^2-a^2}{(a+c)(a+b)}+\frac{a-b}{b+c}$
$\Leftrightarrow \frac{(a-b)^2+(a-c)(b-c)}{ab+bc+ca}\geq\frac{(a-c)(b-c)}{(a+c)(a+b)}+\frac{(a-b)^2}{(a+c)(b+c)}$

$\Leftrightarrow \frac{c^{2}(a-b)^2}{(ab+bc+ca)(a+c)(b+c)}+\frac{a^2(a-c)(b-c)}{(ab+bc+ca)(a+c)(a+b)}\geq0$
BĐT cuối cùng luôn đúng do $c=min\left \{ a,b,c \right \}$

Đẳng thức xảy ra $\Leftrightarrow a=b=c$




#606148 $[\sum a^2+k\sum ab][\sum \frac{1}{(a...

Gửi bởi chardhdmovies trong 30-12-2015 - 18:49

Cho số thực $k\in [-1,2]$, chứng minh BĐT sau đúng với mọi số thực $a,b,c$:
$[a^2+b^2+c^2+k(ab+bc+ca)][\frac{1}{(a-b)^{2}}+\frac{1}{(b-c)^{2}}+\frac{1}{(c-a)^{2}}]\geq \frac{9(2-k)}{4}$




#606112 $\sum \frac{1}{(1+a)^{3}}\g...

Gửi bởi chardhdmovies trong 30-12-2015 - 12:46

cho a,b,c,d là các số thực dương thỏa abcd=1

c/m $\frac{1}{(1+a)^{3}}+\frac{1}{(1+b)^{3}}+\frac{1}{(1+c)^{3}}+\frac{1}{(1+a)^{3}}\geq \frac{1}{2}$

Áp dụng BĐT Holder, ta có: $(1+a)^{3}\leq (1+1)(1+\frac{a\sqrt{a}}{b\sqrt{b}})(1+ab\sqrt{ab})$

Suy ra: $\frac{1}{(1+a)^{3}}\geq \frac{b\sqrt{b}}{2(a\sqrt{a}+a\sqrt{b})(1+ab\sqrt{ab})}$

Tương tự, suy ra: $\frac{1}{(1+b)^{3}}\geq \frac{a\sqrt{a}}{2(a\sqrt{a}+a\sqrt{b})(1+ab\sqrt{ab})}$

Cộng lại ta có: $\frac{1}{(1+a)^{3}}+\frac{1}{(1+b)^{3}}\geq \frac{1}{2(1+ab\sqrt{ab})}$
Thiết lập BĐT tương tự với $c$ và $d$, cộng lại ta có:
$VT\geq \frac{1}{2(1+ab\sqrt{ab})}+\frac{1}{2(1+cd\sqrt{cd})}=\frac{1}{2(1+ab\sqrt{ab})}+\frac{ab\sqrt{ab}}{2(ab\sqrt{ab}+1)}=\frac{1}{2}$

Đẳng thức xảy ra khi và chỉ khi $a=b=c=d=1$




#605330 $P=\frac{a^2}{a^2+b}+\frac{b^2}...

Gửi bởi chardhdmovies trong 26-12-2015 - 15:56

cho mình hỏi sao lại biết cách đặt như vậy? phương pháp là gì?

Bản chất là đồng bậc bđt thôi bạn, bđt thuần nhất thường dễ làm hơn.




#605290 $P=\frac{a^2}{a^2+b}+\frac{b^2}...

Gửi bởi chardhdmovies trong 25-12-2015 - 22:46

 

Cho a,b dương. Tìm GTNN của biểu thức:

$P=\frac{a^2}{a^2+b}+\frac{b^2}{b^2+a}+\frac{1}{12}(\frac{1}{a^3}+\frac{1}{b^3})$

 

Ta sẽ chứng minh: $P\geq \frac{7}{6}$

$\forall a,b>0,\exists x,y,k>0:a=k\sqrt[3]{\frac{x}{y}},b=k\sqrt[3]{\frac{y}{x}}$. Thay vào ta có:
$P=\frac{kx}{kx+y}+\frac{ky}{ky+x}+\frac{1}{12k^{3}}(\frac{x}{y}+\frac{y}{x})$

$P=(\frac{kx}{kx+y}-\frac{k}{k+1})+(\frac{ky}{ky+x}-\frac{k}{k+1})+\frac{1}{12k^{3}}(\frac{x}{y}+\frac{y}{x}-2)+(\frac{2k}{k+1}+\frac{1}{6k^3})$

$P=(x-y)^2[\frac{k(k-1)}{(k+1)(kx+y)(ky+x)}+\frac{1}{12k^{3}xy}]+\frac{2k}{k+1}+\frac{1}{6k^3}$

Xét hàm thấy $\frac{2k}{k+1}+\frac{1}{6k^3} \geq\frac{7}{6}$
Vậy cần chứng minh:

$\frac{k(k-1)}{(k+1)(kx+y)(ky+x)}+\frac{1}{12k^{3}xy}\geq 0$

Với $k\geq1$, bđt hiển nhiên đúng.

Xét $k<1$, áp dụng bđt Cauchy-Schwarz ta có:

$\frac{k(k-1)}{(k+1)(kx+y)(ky+x)}+\frac{1}{12k^{3}xy}\geq \frac{k(k-1)}{(k+1)^3xy}+\frac{1}{12k^{3}xy}$

Vậy cần chứng minh:
$(k+1)^3+12k^4(k-1)\geq0$

Ta có: $(k+1)^3+12k^4(k-1)>(k+1)^3+12k^2(k-1)\geq(k+1)^3-12\frac{(1-k+k+k)^3}{27}=\frac{5}{9}(k+1)^3>0$

Vậy, GTNN của $P$ là $\frac{7}{6}$, đạt được khi $a=b=1$




#597718 $\sum \frac{2a}{2a+b+c}\geq 1+\f...

Gửi bởi chardhdmovies trong 10-11-2015 - 20:21

Cho $a,b,c>0$. C/m:
$\sum \frac{2a}{2a+b+c}\geq 1+\frac{9abc}{2(a+b+c)(ab+bc+ca)}$
P/s: Hy vọng lần này đề không sai




#592964 chọn đội tuyển trường chuyên Nguyễn Du-Đaklak 2015-2016(vòng 2)

Gửi bởi chardhdmovies trong 09-10-2015 - 22:40

Bài 5 (3 điểm)

Tìm tất cả các nghiệm nguyên dương $(x,y,z)$ của phương trình

$\left | 2^x-y^z \right |=1$

Từ giả thiết ta có: $2^{x}=y^z+1$ hoặc $2^x=y^z-1$. Dễ thấy $y$ lẻ
TH1:  $2^{x}=y^z+1$

Nếu $x=1 \Rightarrow y=1,z=1$

Nếu $y=1 \Rightarrow x=1, z\in\mathbb{N}^{*}$

Nếu $z=1 \Rightarrow y=2^x-1, x\in\mathbb{N}^{*}$

Xét $x>1,y>1,z>1$

Nếu đặt $z=2^{a}t$, $t$ lẻ, $a>0,t>0$, ta có:

$2^{x}=(y^{2^{a}}+1)[(y^{2^{a}})^{t-1}-(y^{2^{a}})^{t-2}+...-y^{2^{a}}+1]$
Giả sử $t>1$, do $y^{2^{a}}$ và $t$ lẻ, lớn hơn $1$ nên $(y^{2^{a}})^{t-1}-(y^{2^{a}})^{t-2}+...-y^{2^{a}}+1$ lẻ và lớn hơn $1$, suy ra phương trình vô nghiệm. Do đó $t=1$.

Suy ra $2^{x}=y^{2^{a}}+1 \Leftrightarrow 2^{x}=(y^{2^{a-1}})^{2}+1$

Dễ thấy phương trình vô nghiệm.

TH2: $2^{x}=y^z-1$. Dễ thấy $y>1$

Nếu $x=1 \Rightarrow y=3, z=1$

Nếu $z=1 \Rightarrow y=2^{x}+1, x\in\mathbb{N}^{*}$

Xét $x>1,y>1,z>1$

Chứng minh tương tự suy ra: $z=2^{b}, b\geq1$

Đặt $y^{2^{b-1}}=c,c>1$, ta có: $2^{x}=(c+1)(c-1)$

Đặt $c+1=2^{e}, c-1=2^{f}, e>f>0$

Suy ra $2^{e-1}-2^{f-1}=1$, suy ra: $f<2$

Nếu $f=0 \Rightarrow 2^{x}=3$

Nếu $f=1 \Rightarrow x=3,y=3,z=2$

Vậy, phương trình có các nghiệm $(3,3,2),(k,2^k+1,1),(n,2^n-1,1),(1,1,l)$ với $\forall k\geq1,n\geq1, l>1$




#592928 chọn đội tuyển trường chuyên Nguyễn Du-Đaklak 2015-2016(vòng 2)

Gửi bởi chardhdmovies trong 09-10-2015 - 21:27

Bài 6 (4 điểm)

Cho tập hợp $\mathcal{X}=\left \{ \overline{a_6a_5a_4a_3a_2a_1}\mid 9\ge a_6\ge a_5\ge a_4\ge a_3\ge a_2\ge a_1,a_6\neq 0,a_i\in \mathbb{N},(i=\overline{1,6})\right \}$

$a)$ Tập hợp $\left | \mathcal{X} \right |$ có bao nhiêu phần tử $?$

$b)$ Nếu viết tất cả các số của $\mathcal{X}$ thành một dãy tăng.Tính số hạng thứ $2015$ của dãy đó

a)Bổ đề: Số nghiệm nguyên không âm của bất phương trình:
$\sum_{i=1}^{k}x_{i}\leq m$ là $\binom{m+k}{k}$

Trở lại bài toán:

Đặt $a_{i}=\sum_{k=1}^{i}x_{k}$. Ta có:$x_{i}\geq0, \forall i$ và $\sum_{i=1}^{6}x_{i}\leq 9, (1)$

Nếu $a_{6}=0$ thì $a_{6}=a_{5}=a_{4}=a_{3}=a_{2}=a_{1}=0$

Đặt số nghiệm nguyên không âm của $(1)$ là $S$ thì ta có $|X|=S-1$

Áp dụng bổ đề, ta có: $|X|=\binom{9+6}{6}-1=5004$

 

b) Đặt  $T_{i}$ là số các số thuộc $X$ mà $a_{6}=i, i=1,2,..,9$
Khi đó $T_{i}$ là số nghiệm nguyên không âm của bất phương trình $\sum_{k=1}^{5}x_{k}\leq i$

Áp dụng bổ đề, suy ra: $T_{i}=\binom{i+5}{5}$

Ta có: $\sum_{i=1}^{7}T_{i}=1715 < 2015 < \sum_{i=1}^{8}T_{i}=3002 \Rightarrow a_{6}=8$

$2015-1715=300$
Tương tự, đặt $K_{i}$ là số các số thuộc $X$ mà $a_{6}=8, a_{5}=i, i=0,1,2,..,9$

Áp dụng bổ đề, suy ra: $K_{i}=\binom{i+4}{4}$

Ta có: $\sum_{i=0}^{5}K_{i}=252 < 300 < \sum_{i=0}^{6}K_{i}=462 \Rightarrow a_{5}=6$

$300-252=48$

Đặt $L_{i}$ là số các số thuộc $X$ mà $a_{6}=8, a_{5}=6, a_{4}=i, i=0,1,2,..,9$

Áp dụng bổ đề, ta có: $L_{i}=\binom{i+3}{3}$

Ta có: $\sum_{i=0}^{3}L_{i}=35 < 48 < \sum_{i=0}^{4}L_{i}=70 \Rightarrow a_{4}=4$

Suy ra, số hạng thứ $1715+252+35+1=2003$ là $864000$

Suy ra, số hạng thứ $2015$ là $864320$




#589704 Giải phương trình: $4^{x^{2}}+2^{x}=3^...

Gửi bởi chardhdmovies trong 18-09-2015 - 22:04

Giải phương trình: $4^{x^{2}}+2^{x}=3^{x^{2}}+3^{x}$




#543763 Tổng hợp các định lý trong hình học và ứng dụng của chúng qua các bài toán

Gửi bởi chardhdmovies trong 11-02-2015 - 16:13

Bài toán 2: (IMO 2011) Cho tam giác $ABC$ nội tiếp đường tròn $(O)$. Gọi $l$ là tiếp tuyến bất kỳ của $(O)$. Kí hiệu $l_a, l_b, l_c$ là đường thẳng đối xứng với $l$ qua $BC, CA, AB$ $l_a, l_b, l_c$ cắt nhau tạo thành tam giác $A'B'C'$. Chứng minh rằng đường tròn $(A'B'C')$ tiếp xúc với $(O)$

của thầy Nguyễn Văn Linh

Capture.PNG

Capture.PNG

 

NTP




#543762 Tổng hợp các định lý trong hình học và ứng dụng của chúng qua các bài toán

Gửi bởi chardhdmovies trong 11-02-2015 - 16:10

 

Bài toán 1: Cho đường tròn $(O)$ đường kính $AB$. $P, Q$ là hai điểm bất kỳ trên $(O)$ và khác phía với $AB$. Kẻ $QT$ vuông góc với $AB$, $PC, PD$ lần lượt là tiếp tuyến kẻ từ $P$ tới đường tròn đường kính $AT, BT$. Chứng minh rằng: $PC+PD=PQ$

 

 

Capture.PNG

gọi $(\omega _1),(\omega _2)$ lần lượt là đường tròn đường kính $AT,BT$

áp dụng định lí $casey$ cho bốn đường tròn $(\omega _1),(\omega _2),(P,0),(Q,0)$ thì ta dễ dàng có được $PC+PD=PQ$

 

NTP




#543757 Tổng hợp các định lý trong hình học và ứng dụng của chúng qua các bài toán

Gửi bởi chardhdmovies trong 11-02-2015 - 15:29

 

 

Bài toán 1: Cho đường tròn $(O)$ đường kính $AB$. $P, Q$ là hai điểm bất kỳ trên $(O)$ và khác phía với $AB$. Kẻ $QT$ vuông góc với $AB$, $PC, PD$ lần lượt là tiếp tuyến kẻ từ $P$ tới đường tròn đường kính $AT, BT$. Chứng minh rằng: $PC+PD=PQ$

 

Bài toán 2: (IMO 2011) Cho tam giác $ABC$ nội tiếp đường tròn $(O)$. Gọi $l$ là tiếp tuyến bất kỳ của $(O)$. Kí hiệu $l_a, l_b, l_c$ là đường thẳng đối xứng với $l$ qua $BC, CA, AB$ $l_a, l_b, l_c$ cắt nhau tạo thành tam giác $A'B'C'$. Chứng minh rằng đường tròn $(A'B'C') tiếp xúc với $(O)$

mong bạn ghi lại số bài cho phù hợp với nội dung ban đầu mà chủ topic đề ra

Bài tập 2:(Moldova TST 2010)

Cho tam giác $ABC$ nhọn có $H$ là trực tâm và $M$ là trung điểm $BC$.Kẻ đường thẳng qua $H$ vuông góc với $HM$ và cắt $AB,AC$ lần lượt tại $P,Q$.CMR $MP=MQ$

những lời giải ở trên mình thật sự ấn tượng và sau đây lời giải bằng cách sử dụng định lí con bướm

Capture.PNG

vẽ đường tròn $(M,\frac{BC}{2})$.Gọi $X,Y$ là giao điểm của $PQ$ với đường tròn $(M)$

vì $MH\perp XY\Rightarrow MX=MY$ nên theo định lí con bướm $MP=MQ$

 

 

NTP




#543578 Tổng hợp các định lý trong hình học và ứng dụng của chúng qua các bài toán

Gửi bởi chardhdmovies trong 09-02-2015 - 21:40

Định lí 3:ĐỊNH LÍ CON BƯỚM

Dạng đường tròn:Cho đường tròn $(O)$ với dây cung $AB$ và $I$ là trung điểm $AB$.Qua $I$ vẽ hai dây cung $MN$ và $PQ$ sao cho $E=PM\cap AB,F=QN\cap AB$.CMR $I$ là trung điểm $EF$

Capture.PNG

gọi $C,D$ lần lượt là hình chiếu của $(O)$ trên $PM,NQ$

vì $\Delta PIM\sim \Delta NIQ\Rightarrow \Delta PIC\sim \Delta NID\Rightarrow \widehat{PCI}=\widehat{NDI}$

dễ thấy hai tứ giác $IECO,IFDO$ là các tứ giác nội tiếp $\Rightarrow \widehat{IOE}=\widehat{ICE}=\widehat{IDF}=\widehat{IDF}$

do đó $\Delta OEF$ cân tại $O$ do đó $IE=IF$ 

Capture.PNG

Dạng đường thẳng:Cho tam giác $ABC$, $I$ là trung điểm $BC$.Qua $I$ kẻ đường thẳng $d_1$ cắt $AC, AB$ lần lượt tại $M,N$ và đường thẳng $d_2$ qua $I$ cắt $CA, BA$ tại $P, Q$. Đường thẳng $PN$ cắt cạnh $BC$ tại $E$ và đường thẳng $QM$ cắt cạnh $BC$ tại $F$. CMR $I$ là trung điểm $EF$

xem câu $3.1$ ở đây

Bài tập 2:(Moldova TST 2010)

Cho tam giác $ABC$ nhọn có $H$ là trực tâm và $M$ là trung điểm $BC$.Kẻ đường thẳng qua $H$ vuông góc với $HM$ và cắt $AB,AC$ lần lượt tại $P,Q$.CMR $MP=MQ$

 

NTP